HELPP QUICK PLEASE please

HELPP QUICK PLEASE Please

Answers

Answer 1

Hi! I hope you're enjoying your day!

Please remember that an arithmetic sequence is a sequence where we use either addition or subtraction to get to the next term.

Now, let's find an arithmetic sequence here.

Is it Option A?

Remember: It should have a common difference of 2.

Which means we either add or subtract 2 to get to the next term.

Well, it seems like Option A is right.

Let's check the other options.

Is Option B right? Nope.

Here's why.

Hint: Pay attention to the signs.

We have

1, -3, 5, -7, 9...

So Option B is wrong.

Option C is correct, because we subtract 2 each time.

Option D is wrong, because we multiply by 2 each time.

Hence, the right options are

[tex]\boxed{\boxed{\bold{Options~A~and~C.}}}[/tex]

Hope everything is clear.

If you have any questions, please comment.

#LearningIsFun

[tex]\rule{200}{2}[/tex]


Related Questions

70 points! Brainliest for whoever gets it right! Thanks! Dean ran 2.3 fewer kilometers than Sam. If Dean ran 6.8 km, how far did Sam run? A 2-column table with 4 rows. Column 1 is labeled Situation with entries increasing, difference, finding part of a total, sharing or grouping. Column 2 is labeled Operation with entries +, minus, times, divided by. Select all that apply. You know the difference in the distances the boys ran, so this is a subtraction problem. You are finding the total distance the boys ran, so this is an addition problem. Dean ran part of the distance Sam ran, so this is a multiplication problem. The correct equation is s + 2.3 = 6.8. The correct equation is s – 2.3 = 6.8. The correct equation is 2.3s = 6.8.

Answers

Answer:

→ Distance run by dean = 6.8 km .

also,

→ Distance run by dean = 2.3 fewer kilometers than Sam.

Let us assume that, distance run by same is s km .

so,

→ Distance run by dean = s - 2.3

therefore,

→ s - 2.3 = 6.8

→ s = 6.8 + 2.3

→ s = 9.1 km .

Hence, The correct equation is s – 2.3 = 6.8

Step-by-step explanation:

Answer:

A and E please brainly

Step-by-step explanation:

find the following number of LCM in division method.

a) 6,8 and 12

Answers

Answer:

24 is the LCM.

Step-by-step explanation:

6, 12, 18, 24

8, 16, 24

12, 24

Annie have 56 stamps that are 26 less than one half of my stamps how many stamps do i have?​

Answers

Answer:

You have 164 stamps.

Step-by-step explanation:

Let the no. of my stamps be 'x'

Half of x subtracted by 26 = 56

[tex]\frac{1}{2} x[/tex] - 26 = 56

[tex]\frac{1}{2} x[/tex] = 56 + 26 =  82

[tex]x[/tex] = 82 x 2 =  164

Let's check

[tex]\frac{164}{2}[/tex] - 26 = 56

[tex]\frac{164}{2}[/tex] = 56 + 26

164     = 82 x 2

164 = 164!

The answer is correct.

Imani was assigned to make 64 cookies for the bake sale. If Imani made 25% more than they were assigned, how many cookies were made

Answers

Answer:

80 cookies

Step-by-step explanation:

25% of 64 is 16
16+64=80

Imani made total of 8 cookies.

What is Algebra?

A branch of mathematics known as algebra deals with symbols and the mathematical operations performed on them.

Variables are the name given to these symbols because they lack set values.

In order to determine the values, these symbols are also subjected to various addition, subtraction, multiplication, and division arithmetic operations.

Given:

Imani was assigned to make 64 cookies for the bake sale.

let she made x cookies.

So, x = 64 + 25% of 64

x = 64 + 25/100 (64)

x = 64 + 64/4

x = 64 + 16

x = 80

Hence, the total cookies made is 80.

Learn more about algebra here:

https://brainly.com/question/24875240

#SPJ2

If f(x)=3x-1 and g(x)=x+5, find (f o g)(x) and (g o f)(x)

(f o g)(x)=

(g o f)(x)

Answers

Answer:

(f o g)(x)= 3x + 14

(g o f)(x) = 3x + 4

Step-by-step explanation:

(f o g)(x) = f(g(x)) = f(x + 5) = 3(x + 5) - 1 = 3x + 15 - 1 = 3x + 14

(g o f)(x) = g(f(x)) = g(3x - 1) = 3x - 1 + 5 = 3x + 4

Answer:

[tex](f \circ g)(x) = 3x + 14[/tex]

[tex](g \circ f)(x) = 3x + 4[/tex]

Step-by-step explanation:

Hello!

Rewrite the equations:

[tex](f \circ g)(x) = f(g(x))[/tex][tex](g \circ f)(x) = g(f(x))[/tex]

Given that:

f(x) = 3x - 1g(x) = x + 5

Solve for [tex](f \circ g)(x)[/tex]:[tex]f(g(x)) = 3(g(x)) - 1[/tex][tex]f(x + 5) = 3(x + 5) - 1[/tex][tex]f(x + 5) = 3x + 15 - 1[/tex][tex]f(x + 5) = 3x + 14[/tex]

[tex](f \circ g)(x) = 3x + 14[/tex]

Solve for [tex](g \circ f)(x)[/tex]:[tex]g(f(x)) = f(x) + 5[/tex][tex]g(3x - 1) = (3x - 1) + 5[/tex][tex]g(3x - 1) = 3x +4[/tex]

[tex](g \circ f)(x) = 3x + 4[/tex]

Subtract 4 from 8. Then, multiply by z.

Answers

Answer:

Kinda vague but here...

Step-by-step explanation:

z*(8-4)
z*(4)
4z

What annual interest rate will cause your money to double in four years

Answers

Answer:

The rule also means if you want your money to double in 4 years, you need to find an investment that earns 18% per year compounded annually.

Step-by-step explanation:

Write as a
single fraction
X-2
X+ 3
X+1
x-11

Answers

Answer:

x-2/1

X+3/1

x+1/1

x-11/1

Step-by-step explanation:

just put over 1 in the bottom for all the equation

Part 1: Rewrite the mathematical expressions. a) 4³=​

Answers

Answer:

64

Step-by-step explanation:

Each wall in the room is 10feet by 11 feet how many rolls of wall paper will you need of each roll covers 25 square feet

Answers

Answer:

4.4 rolls

Step-by-step explanation:

10 x 11 = 110

110 / 25 = 4.4

Hi can anyone help? Is this 12?

Answers

Answer: Yes

Step-by-step explanation:

∛1728= ∛(2 x 2 x 2) x (2 x 2 x 2) x (3 x 3 x 3)

= 2 x 2 x 3

=12

Solve Calculate the product: 2/9 × 63. A. 7 B. 14 C. 18 D. 45

Answers

Answer:

14

Step-by-step explanation:

You can solve it like this:

63x2=126

126/9=14


What is the volume of this container? ?
2 ft2
2 ft
2 ft²
I ft
4 ft²
I ft
4 ft3

Answers

Answer:

2ft^3

Step-by-step explanation:

Volume= L× W× H

[tex]volume = 1 \times 1 \times 2[/tex]

[tex]volume = 2ft^{3} [/tex]

~

Samantha has a shelf that is 95 over 4 inches wide. How many books can Samantha arrange on the shelf if each book is 5 over 4 inches thick? Work it out.

Answers

Answer:

19 books

Step-by-step explanation:

95/4÷5/4=19bks

Which of the following figures is a pair of lines that are perpendicular? ​

Answers

Answer:

D is the correct answer.

Step-by-step explanation:

The lines are perpendicular as they go thiugh eachother at the center.

Answer:

4.) D

Step-by-step explanation:

A ( I'm not sure how to explain A )

B is an acute angle

C is a parallel line

D is a perpendicular line

I hope this helps!

The length of a cell phone is 1.2 inches and the width is 3.4 inches. The company making the cell phone wants to make a new version whose length will be 1.68 inches. Assuming the side lengths in the new phone are proportional to the old phone, what will be the width of the new phone?

Answers

The length of a cell phone is 1.2 inches and the width is 3.4 inches. The company making the cell phone wants to make a new version whose length will be 1.68 inches. Assuming the side lengths in the new phone are proportional to the old phone, what will be the width of the new phone?

The Answer is : 4.76

WILL MARK AS BRAINLIEST!!

On the first day of school, the teacher places 3 pencils and 2 highlighters on
each desk. Which shows the ratio of highlighters to pencils? Select THREE
ratios.
A. 3 to 2
B. 2:3
C. 2 to 3
D. 3/2
E. 3:2
F. 2/3

Answers

Answer:

B, C, and F

Step-by-step explanation:

A woman acquires a bond that has maturity date of 10 years and a face value of $5,000. If the bond has a coupon rate of 6%, then how much interest will she earn each year? *

Answers

Answer:

I = $3000

Step-by-step explanation:

I = PRT/100

P = principle= $5000

R = rate = 6%

T = time ( yrs) = 10 yrs

I = (5000*6*10) / 100

I = $3000

here a subtraction problem that has been solved incorrectly for 44-16=28 choose the addition sentence that check this subtraction problem the choices are 28+16=32 28+44=72 28+16=44 which one is correct​

Answers

Answer:

Answer:

28+16=44

Step-by-step explanation:

point of correction it has been solved correctly

when subtracting 44 - 16 it gives you 28

=> 44 - 16

=> 28

SO IN ORDER TO CHECK IF THE PROBLEM WAS SOLVE CORRECTLY

you will minus 28 from one of the numbers before the equal sign

the correct option is 28 + 16 equals to 44

=> 28+16=44

Answer:

28 + 16 = 44

Step-by-step explanation:

basically it is like isolating the biggest number (44) by bringing 16 to the right side of the equation. from 44 - 16 = 28 to 44 = 28 + 16 which is the same as 28 + 16 = 44

23. Which number below could not be used to


PLEASEE HELP!!

Answers

0.75 i think so lol HI

Round 34,528 to the nearest hundred. Write the comma (",") in your final answer.

Answers

Answer:

34,500

Step-by-step explanation:

34,528 rounds down rather than up because 528 is closer to 500 than 600 so it will round down to 34,500

Answer:

34,500

Step-by-step explanation:

34,528 = 34,500 because when you round the 5 in the hundred place the 2 isn't high enough to make the 5 round up so its 34,500.

5-10 round up 1-4 round down.

Rename 2/3 and 3/4 using the least common denominator.

Answers

Answer:

17/12

Step-by-step explanation:

because it 12 the most minimum amounted common multiple of 3 and 4


Answers : 8/12 and 9/12

2/3 3/4
Looking at the denominators their least common multiple is 12
Hence the two denominators should be 12

2/3 = 2/3 x 4/4 = 8/12

3/4 = 3/4 x 3/3 = 9/12

Multiply.
5 - 6
3
2
1
0
-10

Answers

Answer:

Hello! the answer is negative 1

Step-by-step explanation:

5 - 6 is negative 1

there's no answer option for it

_AnnieTheDreamGirl, 3/8/2022

I think you meant Subtract or multiply?

Answer:

-1

30

Step-by-step explanation:

5  - 6 = -1

The first number is least than the second number thus, it a negative..

----------------------------------------------------------------------------------------------------------

5 * 6 = 30

----------------------------------------------------------------------------------------------------------

As shown it none of the answer choices.. I think there a mistaken with your question..

[RevyBreeze]

Which of the numbers below are common
multiples of 2 and 5? Select all that apply.
A) 15
B) 20
C) 30
D) 35
E) 50

Answers

Answer:

20, 30, 50, they all go into 2 and they also go into 6, two doesn't divide equally into 5.

5 used books for 9.95

Answers

Answer:your mom

Step-by-step explanation:

PLEASE SHOW HOW YOU GOT THE ANSWER THANKS!
GIVING BRAINLIEST!!!

Answers

Answer:

a) 5  b) 1  c) 0  d) -4

Explanation:

a) 10[(1/2+1/4) + 2(1/8)] ÷ 2

Ok, there's a lot going on. You will want to follow PEMDAS, the acronym of the Order of Operations. It stands for Parentheses, Exponents, Multiplication/Division, and Addition/Subtraction. The M/D can be switched depending on what comes first from left to right, and same goes for A/S.

First, start off by doing everything in the brackets.

Let's add 1/2 and 1/4. To add fractions, they have to have a common denominator. I'm making 1/2 into 2/4 but multiplying the top and bottom by 2. Now it is 2/4 + 1/4 so we can add to get 3/4.

The other thing inside the brackets is 2(1/8). We can just multiply straight across and get 2/8. This can be simplified to 1/4.

Now, we add 3/4 and 1/4. The denominators are already the same, so we can just add them together to get 4/4 or 1.

Rewriting the equation, we now have 10(1)÷2

10•1 = 10

10 ÷ 2 = 5

The answer to question a is 5.

b) √(0.6)² + (0.8)²

First, we need to square both terms. Squaring is the same as multiplying the number by itself.

0.6 • 0.6 = 0.36

0.8 • 0.8 = 0.64

Add them together: 0.36 + 0.64 = 1

√1 = 1

The answer to question b is 1.

c) (1/5 - 3/5) • √6•3/2 + (√36 ÷ √5²)

Let's start with the first bit: (1/5 - 3/5)

The denominators are the same, so just subtract and you get -2/5

The second bit now: [tex]\sqrt{6 *\frac{3}{2} }[/tex]

Multiply across and you get 18/2, which can be simplified to 9

√9 = 3

The third section: ([tex]\sqrt{36}[/tex]÷ [tex]\sqrt{5^{2} }[/tex])

The square root of 36 = 6 (because 6•6=36, so it's right)

The square root and the square cancel each other out, so the second part is 5.

6 ÷ 5 = 1.2

Now put it all together: -2/5 • 3 + 1.2

I'm making the 1.2 into a fraction so it is the same as the fraction: 1.2 = 6/5

We now have -2/5 • 3 + 6/5

First we multiply -2/5 and 3 across:

-2/5 • 3 = -6/5

Now we add -6/5 and 6/5 across:

-6/5 + 6/5 = 0

d) [-1.5 + √0.25 - (-0.75)] • 2^4

Doing the brackets first:

The square root of 0.25 = 0.5 (because 0.5 • 0.5 = 0.25)

-1.5 + 0.5 - -0.75

-1 + 0.75 (subtracting a negative make a positive)

-0.25

-0.25 • 2^4

2^4 = 16 (this is saying 2•2•2•2 which is 16)

-.25 • 16 = -4

What number is 9 times as much as 400 ? A 391 B 409 C 3,600 D 3,609​

Answers

Answer:

C.3600

...............

The number which is 9 times as much as the number 400 is number 3600. The option C is the correct option.

What is n times of a number?

The meaning of n times of a number is that, how many times a number n is required to bring the result as the other number.

The number which is 9 times as much as 400 has to be found out. Let suppose this number is n.

To find the value of n, we have to multiply the number 400 with the number 9. Thus,

[tex]n=400\times 9\\n=3600[/tex]

The value of n is 3600.

Thus, the number which is 9 times as much as the number 400 is number 3600. The option C is the correct option.

Learn more about the multiplication of two number here;

https://brainly.com/question/18248493

help me with this one please :) thank you​

Answers

If it needs a sol with final number
Here we go
4*(4^2)_3*(4)+6
=4*16_12+6
=64_12+6
=58

?The measure of chance variation

1 - Standard error
2- Standard deviation
3- Variance​

Answers

Answer of these questions is Mean

4 > 3 is an example of an?

Answers

Answer:

is an example of enequality

Other Questions
Over 14 million Americans are unemployed. Now imagine you're one of them. Your savings are gone. You've lost your house. And you're down to your last $1,000. Can you make it through the month? 1. How do you feel about your situation? Do you think you can live 30 days with $1,000? Explain. what dose a compress pictures command do ? define Brownian motion 408 takes the bus that's 60 percent of the kids at school what is the told number of kids at school can someone explain this to me please? Need help on this question ASAP pleasee please! Report your answer using appropriate number of significant figures. Moels of water a. 9.52 x 1026 molecules ____mol When you write your paper, be sure to answer these questions. (Use these questions as guides for writing, but don'tactually write out the questions and then answer them.)1. How does the condition develop?2. When does it develop?3. Do certain races, a certain gender, or certain types of people get it more often than others?4. What does it look like in a person?5. What are the limitations a person has?6. Is prevention possible?7. What treatments are available, if any?8. How do people manage and cope with the condition?9. How can we be sensitive to people with this type of physical challenge? can someone help me please Mr Dean and mr. Mack each make a box plot to represent the grades from the same unit test for their first period classes 1. The number of students in mr Mack first period that scored between 78 and 90 on unit test represent what percent of his students.A.25%B.50%C.75%D.100% Think About a Plan The gazebo in the photo is built in the shape of a regular octagon. Each side is 8 ft long, and the enclosedarea is 310.4 ft2. What is the length of the apothem? How can you draw a diagram to help you solve the problem? How can you use the area of a regular polygon formula? HELPPPP i was absent this is a missing work and I'm trying to finish as soon as possible to make my parents happy about my grades An area that has the power to make, implement, and enforce laws.. A student heats the same amount of two different liquids over Bunsen burners. Each liquid is at room temperature when the student begins. If Liquid 1 has a higher specific heat than Liquid 2, then Liquid 1 will evaporate sooner. evaporate sooner. take longer to increase in temperature. take longer to increase in temperature. condense over a longer period. condense over a longer period. chemically break down at a faster rate. MARKING BRAINLY, HELP PLEASE!! Emmanuel is taking still-life photographs of fruit. He is concerned about distractions in the background of his photos. What could Emmanuel do to help reduce these background distractions?A. Use a wide-angle lens.B. Move in closer to his subject.C. Get further away from his subject.D. Add additional background elements. Decide which of the following is a reasonable answer for 7/5 x 7. pls help7/579 4/5 Bisphenol A is a fat soluble chemical and can get stored in the fat cells of humans. Steven henteges , a spokesman for the American chemistry council said the new data on a consumer exposure to BPA in Canada is very reassuring. Do you think this statement is reassuring? Explain your reason. (Help me pleaseee!) URGENTLY can someone help me with this Find the volume of the prism.A) 216 kmB) 109 kmC) 113 kmD) 152 km Please help!The atoms that undergo nuclear fusion become smallerTrueFalse